Vous êtes sur la page 1sur 14

Resource Management 4.2.

1
4-2 TRANSPORTATION

INTRODUCTION
A scooter production company produces scooters at the units situated at various places (called origins)
and supplies them to the places where the depot (called destination) are situated.
Here the availability as well as requirements of the various depots are finite and constitute the limited
resources.
This type of problem is known as distribution or transportation problem in which the key idea is to
minimize the cost or the time of transportation.

MATHEMATICAL FORMULATION OF TRANSPORTATION PROBLEM
Some Definitions
The following terms are to be defined with reference to the transportation problems:
(A) Feasible Solution (F.S.): A set of non-negative allocations xij 0 which satisfies the row and
column restrictions is known as feasible solution.
(B) Basic Feasible Solution (B.F.S.): A feasible solution to a m-origin and n-destination problem is
said to be basic feasible solution if the number of positive allocations are (m+n1).
If the number of allocations in a basic feasible solutions are less than (m+n1), it is called degenerate
basic feasible solution (DBFS) (otherwise non-degenerate).
(C) Optimal Solution: A feasible solution (not necessarily basic) is said to be optimal if it minimizes
the total transportation cost.

PRACTICAL PROBLEMS

THE TRANSPORTATION PROBLEM
Q 1:
Depot/ Unit B1 B2 B3 B4 Stock
A1 (2) (3) (5) (1) 8
A2 (7) (3) (4) (6) 10
A3 (4) (1) (7) (2) 20
Requirement 6 8 9 15 38

(All terms are in hundreds)
In order to find the solution of this transportation problem we have to follow the steps given below.
(A) Initial basic feasible solution
(B) Test for optimization.

Let us consider these steps one by one.
(A) Initial Basic Feasible Solution:
(I) North-West corner rule

Transportation 4.2.2
(II) Lowest cost entry method
(III) Vogels approximation method

A1: (A) Initial Basic Feasible Solution
(I) North-West corner rule

Depot/ Unit B1 B2 B3 B4 Stock
A1 6(2) 2(3) (5) (1) 8
A2 (7) 6(3) 4(4) (6) 10
A3 (4) (1) 5(7) 15(2) 20
Requirement 6 8 9 15 38

From the above table, the cost of transportation is calculated:
62 + 23 + 63 + 44 + 57 + 152
= 12 + 6 + 18 + 16 + 35 + 30 = 117 i.e., 11700.
(II) Lowest cost entry method
Depot/ Unit B1 B2 B3 B4 Stock
A1 (2) (3) (5) 8(1) 8
A2 1(7) (3) 9(4) (6) 10
A3 5(4) 8(1) (7) 7(2) 20
Requirement 6 8 9 15 38
From the above facts, calculate the cost of transportation as
81 + 17 + 94 + 54 + 81 + 72
= 8 + 7 + 36 + 20 + 8 + 14= 93 i.e., 9300.

(III) Vogels approximation method
Depot/ Unit B1 B2 B3 B4 Stock RP1 RP2 RP3 RP4
A1 6(2) (3) (5) 2(1) 8 1 2
2
- -
A2 (7) 1(3) 9(4) (6) 10 1 1 1 1
A3 (4) 7(1) (7) 13(2) 20 1 1 1 6
4

Requirement 6 8 9 15 38
CP1 2
1
2 1 1
CP2 - 2 1 1
CP3 - 2 1 4
3

CP4 - 2 3 -
From the above facts calculate the cost of transportation as
62 + 21 + 13 + 94 + 71 + 132
= 12 + 2 + 3 + 36 + 7 + 26 = 86 i.e., 8600.


Resource Management 4.2.3
(B) Test for Optimization
Definition: A basic feasible solution of an (m n) transportation problem is said to be non-
degenerate if it has following two properties:
(a) Initial basic feasible solution must contain exactly m+n1 number of individual allocations.
(b) These allocations must be in independent positions. Independent positions of a set of allocations
mean that it is always impossible to form any closed loop through these allocations.
The following theorem is also helpful in testing the optimality.
Theorem: If we have a B.F.S. consisting of m+n1 independent positive allocations and a set of
arbitrary number u
i
and v
j
(i=1,2,...m; j=1,2,...n) such that c
rs
= u
r
+v
s
for all occupied cells (r,s) then
the evaluation d
ij
corresponding to each empty cell (i, j) is given by
d
ij
= c
ij
(u
i
+v
j
)

Algorithm for optimality test
In order to test for optimality we should follow the procedure as given below:
Step 1: Start with B.F.S. consisting of m+n1 allocations in independent positions.
Step 2: Determine a set of m+n numbers:
u
i
(i=1,2,....m) and v
j
(j=1,2,...n)
such that for each occupied cells (r,s)
c
rs
= u
r
+v
s

Step 3: Calculate cell evaluations (unit cost difference)
d
j
for each empty cell (i,j) by using the formula
d
ij
= c
ij
(u
i
+v
j
)
Step 4: Examine the matrix of cell evaluation d
ij
for negative entries and conclude that
i. If all d
ij
> 0 then Solution is optimal and unique.
ii. (ii)If all d
ij
0 then At least one d
ij
= 0 then Solution is optimal and alternate solution also
exists.
iii. If at least one d
ij
< 0 then Solution is not optimal.
If it is so, further improvement is required by repeating the above process.
Step 5:
i. See the most negative cell in the matrix [d
ij
].
ii. Allocate to this empty cell in the final allocation table. Subtract and add the amount of
this allocation to other corners of the loop in order to restore feasibility.
iii. The value of , in general is obtained by equating to zero the minimum of the allocations
containing (not + ) only at the corners of the closed loop.
iv. Substitute the value of and find a fresh allocation table
Step 6: Again, apply the above test for optimality till you find all d
ij
0

Computation for optimality test from the initial basic feasible solution as obtained by VAM
Step 1: (i) In this table number of allocations = 3+41=6
Step 2: Determine a set of (m+n), i.e., (3+4) numbers u1, u2, u3, and v1, v2, v3, and v4 for each
occupied cells by taking u
1
as an arbitrary constant (say zero) then by using c
ij
= u
i
+v
j
try to find all u
i

and v
j
as

Transportation 4.2.4
B1 B2 B3 B4 u
i

A1 2 1 0
A2 3 4 3
A3 1 2 1
v
j
2 0 1 1

Step 3: Cost matrix for the empty positions
c
ij
u
i
+v
j
d
ij
=[c
ij
][u
i
+v
j
]
B1 B2 B3 B4 u
i
B1 B2 B3 B4 u
i
B1 B2 B3 B4 u
i

A1 3 5 A1 0 1 0 A1 3 4
A2 7 6 A2 5 4 3 A2 2 2
A3 4 7 A3 3 2 1 A3 1 5
v
j
v
j
2 0 1 1 v
j


Step 4: Here all d
ij
> 0 then Solution obtained by Vogels approximation method is an optimal
solution

Q2: Where IBFS is not optimal: Data for the transportation problem

Warehouse/
Factory
W1 W2 W3 W4 Factory
Capacity
F1 (19) (30) (50) (10) 7
F2 (70) (30) (40) (60) 9
F3 (40) (8) (70) (20) 18
Warehouse
Requirement
5 8 7 14 34
Find out the initial basic feasible solution by Vogels approximation method and test optimality.
A2:
Warehouse/
Factory
W1 W2 W3 W4 Factory
Capacity
RP1 RP2 RP3 RP4
F1 5(19) (30) (50) 2(10) 7 9 9 40 40
F2 (70) (30) 7(40) 2(60) 9 10 10 20 20
F3 (40) 8(8) (70) 10(20) 18 12 20 50
3
-
Warehouse
Requirement
5 8 7 14 34
CP1 21 22
1
10 10
CP2 21
2
- 10 10
CP3 - - 10 10
CP4 - - 10 50
4


Resource Management 4.2.5
Cost = 519 + 210 + 740 +260 + 88 +1020 = 779
Step 1: Number of allocations = 3+41=6 and they are in independent position.
Step 2: Assumed v
4
= 0
Warehouse/
Factory
W1 W2 W3 W4 Capacity u
i
F1 (19) (10) 10
F2 (40) (60) 60
F3 (8) (20) 20
Requirement v
i
[v
4
=0] 9 -12 -20 0

Step 3: Cost matrix for the empty positions
c
ij
u
i
+v
j
d
ij
=[c
ij
][u
i
+v
j
]
W1 W2 W3 W4 W1 W2 W3 W4 u
i
W1 W2 W3 W4
F1 30 50 F1 -2 -10 10 F1 32 60
F2 70 30 F2 69 48 60 F2 1 -18
F3 40 70 F3 29 0 20 F3 11 70
v
j
v
j
9 -12 -20 0 v
j

Step 4: Since one d
ij
< 0, therefore, the solution is not optimal.
Step 5:
Warehouse/
Factory
W1 W2 W3 W4 Factory
Capacity
F1 5(19) (30) (50) 2(10) 7
F2 (70) + (30) 7(40) - 2(60) 9
F3 (40) - 8(8) (70) + 10(20) 18
Warehouse
Requirement
5 8 7 14 34
(iii)
Warehouse/
Factory
W1 W2 W3 W4 Factory
Capacity
F1 5(19) (30) (50) 2(10) 7
F2 (70) 2 (30) 7(40) (60) 9
F3 (40) 6 (8) (70) 12(20) 18
Warehouse
Requirement
5 8 7 14 34
This improved basic feasible solution gives the cost for this solution as
5(19)+2(10)+2(30)+7(40)+6(8)+12(20) = 743.

Step 6: Test this improved solution for optimality by repeating steps 1,2,3 and 4. In each step,
following matrices are obtained:

Transportation 4.2.6
Step 1: Number of allocations = 3+41=6 and they are in independent position.
Step 2: Assumed u
3
= 0
Warehouse/
Factory
W1 W2 W3 W4 Capacity u
i
u
3
=0
F1 (19) (10) -10
F2 (30) (40) 22
F3 (8) (20) 0
Requirement v
i
29 8 18 20

Step 3: Cost matrix for the empty positions
c
ij
u
i
+v
j
d
ij
=[c
ij
][u
i
+v
j
]
W1 W2 W3 W4 u
i
W1 W2 W3 W4 u
i
W1 W2 W3 W4
F1 30 50 F1 -2 8 -10 F1 32 42
F2 70 60 F2 51 42 22 F2 19 18
F3 40 70 F3 29 18 0 F3 11 52
v
j
v
j
29 8 18 20 v
j
29 8 18 20
Step 4: Since all d
ij
< 0, therefore, the solution is optimal with minimum cost = 743.

Q3: A manufacturer has distribution centres X, Y, and Z. These centres have 40, 20 and 40 units of
his product. His retail outlets at A, B, C, D and E require 25,10,20,30 and 15 units respectively. The
transport cost in (Rupees/Unit) between each centre and each outlet is given in the following table:
Distribution Centre
\Retail outlets
A B C D E
X 55 30 40 50 40
Y 35 30 100 45 60
Z 40 60 95 35 30
Find out the optimum distribution cost.
A3:
A B C D E Supply RP1 RP2 RP3 RP4 RP5
X (55) 10(30) 20(40) (50) 10(40) 40 10 10
2
10
3
- -
Y 20(35) (30) (100) (45) (60) 20 5 5 10 10 10
5

Z 5(40) (60) (95) 30(35) 5(30) 40 5 5 5 5 5
Demand 25 10 20 30 15 100
CP1 5 0 55
1
10 10
CP2 5 0 - 10 10
CP3 5 - - 10 10
CP4 5 - - 10 30
4

CP5 5 - - 10 -

Cost = 1030 + 2040 + 1040 + 2035 + 540 + 3035 + 530 = 3600

Resource Management 4.2.7
Step 1: Number of allocations = 3+51=7 and they are in independent position.
Step 2: Assumed u
1
= 0
A B C D E u
i
[u
1
=0]
X (30) (40) (40) 0
Y (35) -15
Z (40) (35) (30) -10
v
i
50 30 40 45 40

Step 3: Cost matrix for the empty positions
c
ij
u
i
+v
j
d
ij
=[c
ij
][u
i
+v
j
]
A B C D E u
i
A B C D E u
i
A B C D E
X 55 50 X 50 45 0 X 5 5
Y 30 100 45 60 Y 15 25 30 25 -15 Y 15 75 15 35
Z 60 95 Z 20 30 -10 Z 40 65
v
j
v
j
50 30 40 45 40 v
j

Step 4: Since all d
ij
> 0, therefore, the solution is optimal.

Q4: (Unbalanced Problem) A company requires for the next month 300, 260 and 180 tones of stone
chips for its three constructions C1, C2 and C3 respectively. Stone chips are produced by the
company at three mineral fields. All the available boulders must be crushed into chips. Any excess
chips over the demands at sites C1, C2 and C3 will be sold ex-fields.
The fields are M1, M2 and M3 which will yield 250, 320 and 280 tones of stone chips respectively.
Transportation costs from mineral elds to construction sites vary according to distances, which are
given below in monetary unit (MU).
From\To C1 C2 C3
M1 8 7 6
M2 5 4 9
M3 7 5 5
(i) Determine the optimal plan for the company and the overall transportation cost in MU.
(ii) What are the quantities to be sold from M1, M2 and M3 respectively?
A4:
From\To C1 C2 C3 D1 To RP1 RP2 RP3 RP4
M1 (8) (7) 140(6) 110(0) 250 6
1
1 1 1
M2 300(5) 20(4) (9) (0) 320 4 1 5
3
-
M3 (7) 240(5) 40(5) (0) 280 5 0 0 0
From 300 260 180 110 850
CP1 2 1 1 0
CP2 2
2
1 1 -
CP3 - 1 1 -
CP4 - 2
4
1 -

Transportation 4.2.8
Cost = 1406 + 3005 + 204 +2405 +405 = 3,820
Step 1: Number of allocations = 3+41=6 and they are in independent position.
Step 2: Assumed u
1
= 0
Warehouse/
Factory
W1 W2 W3 W4 Capacity u
i
U
1
=0
F1 (6) (0) 0
F2 (5) (4) 0
F3 (5) (5) 1
Requirement v
i
5 4 6 0

Step 3: Cost matrix for the empty positions
c
ij


u
i
+v
j


d
ij
=[c
ij
][u
i
+v
j
]
W1 W2 W3 W4 u
i
W1 W2 W3 W4 u
i
W1 W2 W3 W4 u
i

F1 (8) (7) F1 5 4 0 F1 3 3
F2 (9) (0) F2 6 0 0 F2 3 0
F3 (7) (0) F3 6 1 1 F3 1 0
v
j
v
j
5 4 6 0 v
j

Step 4: Since all d
ij
> 0, therefore, the solution is optimal.

Q5: Unbalanced, Degenerated and Maximization Problem
Ladies Fashion Shop wishes to purchase the following quantity of summer dresses:
Dress Size I II III IV
Quantity 100 200 450 150
Three manufacturers are willing to supply dresses. The quantities given below are the maximum that
they are able to supply of any given combination of orders for dresses: Manufacturers A, B and C, can
supply total quantity of 150, 450 and 250. The shop expects the profit per dress to vary with the
manufacturer as given below:
Size I II III IV
A 2.5 4.0 5.0 2.0
B 3.0 3.5 5.5 1.5
C 2.0 4.5 4.5 2.5
Required:
a) Use the transportation technique to solve the problem of how the orders should be placed with the
manufacturers by the fashion shop in order to maximize profit.
b) Explain how you know there is no further improvement possible.

A5: Profit Maximization Problem
From\To I II III IV To
A (2.5) (4.0) (5.0) (2.0) 150
B (3.0) (3.5) (5.5) (1.5) 450

Resource Management 4.2.9
C (2.0) (4.5) (4.5) (2.5) 250
D1 (0) (0) (0) (0) 50
From 100 200 450 150 900

Profit maximization problem converted into minimization problem (Highest Profit in the cell Profit
in the cells)
From\To I II III IV To RP1 RP2 RP3
A 100(3) (1.5) (0.5) 50(3.5) 150 1 1.5 .5
B (2.5) (2) 450(0) (4) 450 2
1
- -
C (3.5) 200(1) (1) 50(3) 250 0 2
2
.5
D1 (5.5) (5.5) (5.5) 50(5.5) 50 0 0 0
From 100 200 450 150 900
CP1 .5 .5 .5 .5
CP2 .5 .5 - .5
CP3 .5
3
- - .5

Maximum Profit = 1002.5 + 502 + 4505.5 +2004.5 + 502.5 + 500 = 3,850
Step 1: Number of allocations < 4+41=7 and the problem is de-generated. Hence is introduced in
the least cost empty cell to test its optimality
Step 2: Assumed u
1
= 0
From\To I II III IV u
i

A (3) (0.5) (3.5) 0
B (0) -0.5
C (1) (3) -0.5
D1 (5.5) -2
v
j
3 1.5 0.5 3.5

Step 3: Cost matrix for the empty positions
c
ij
u
i
+v
j
d
ij
=[c
ij
][u
i
+v
j
]
I II III IV To I II III IV u
i
W1 W2 W3 W4 u
i

A 1.5 A 1.5 0 F1 0
B 2.5 2 4 B 2.5 1 3 -0.5 F2 0 1 1
C 3.5 1 C 2.5 0 -0.5 F3 1 1
D1 5.5 5.5 5.5 D1 1 .5 1.5 -2 4.5 5 4
v
j
3 1.5 0.5 3.5 v
j

Step 4: Since all d
ij
> 0, therefore, the solution is optimal.



Transportation 4.2.10
[CWA RTP D11]
Q6: A company has three plants F1, F2 F3 from which it supplies to 4 markets: A, B, C & D.
Determine the optimal transportation plan from the following table giving the plant to market
shipping costs, quantities available at each plant and quantities required at each market.

Plant Markets
Available
at plant A B C D
F
1
13 11 15 20 2
F
2
17 14 12 13 6
F
3
18 18 15 12 7
Requirement 3 3 4 5 15

A6: The problem is balanced transportation problem because the demand and supply both are equal to
15. Now obtain the initial feasible solution by Vogels method. The cost matrix and the penalties for
given problem are as under:

From\To A B C D A RP1 RP2 RP3
F
1
2(13) (11) (15) (20) 2 2 - -
F
2
(17) 3(14) 3(12) (13) 6 1 1 1
F
3
1(18) (18) 1(15) 5(12) 7 3 *3 3
R 3 3 4 5 15
CP1 4
1
3 3 1
CP2 1 4
2
3 1
CP3 1 - *3
3
1

*In CP3, column 3 and in RP3 row 3 has same penalty and cost in its concerned column or row also
12. Hence column 3 in CP3 is taken for arbitrary assignment.
Minimum Cost = 213+314+312+118+115+512 = 197

Step 1: Number of allocations is equal to m+n-1 [4+31 = 6] and the problem is generated.
Step 2: Assumed u
1
= 0
From\To A B C D u
i

F
1
(13) 0
F
2
(14) (12) 2
F
3
(18) (15) (12) 5
v
j
13 12 10 7



Resource Management 4.2.11
Step 3: Cost matrix for the empty positions
c
ij
u
i
+v
j
d
ij
=[c
ij
][u
i
+v
j
]
A B C D To A B C D u
i
A B C D u
i

F
1
11 15 20 F
1
12 10 7 0 F
1
-1 5 13
F
2
17 13 F
2
15 9 2 F
2
2 4
F
3
18 F
3
17 5 F
3
1
v
j
13 12 10 7 v
j


Step 4: Since all d
ij
is not positive, therefore, the solution is not optimal.
As the cell [F
1
.A] is negative, it is the new occupied cell
Step 5:
F\T A B C D A F\T A B C D A
F
1
- 2(13) + (11) (15) (20) 2 F
1
1(13) 1(11) (15) (20) 2
F
2
(17) - 3(14) + 3(12) (13) 6 F
2
(17) 2(14) 4(12) (13) 6
F
3
+ 1(18) (18) - 1(15) 5(12) 7 F
3
2(18) (18) (15) 5(12) 7
R 3 3 4 5 15 R 3 3 4 5 15

This improved basic feasible solution gives the cost for this solution as
Minimum Cost = 113+111+214+412+218+512 = 196

Step 6: Test this improved solution for optimality by repeating steps 1, 2, 3 and 4. In each step,
following matrices are obtained
Step 1: Number of allocations is equal to m+n-1 [4+31 = 6] and the problem is generated.
Step 2: Assumed u
1
= 0
From\To A B C D u
i

F
1
(13) (11) 0
F
2
(14) (12) 3
F
3
(18) (12) 5
v
j
13 11 9 7

Step 3: Cost matrix for the empty positions
c
ij
u
i
+v
j
d
ij
=[c
ij
][u
i
+v
j
]
A B C D To A B C D u
i
A B C D u
i

F
1
15 20 F
1
9 7 0 F
1
6 13
F
2
17 13 F
2
16 10 3 F
2
1 3
F
3
18 15 F
3
16 14 5 F
3
2 1
v
j
13 11 9 7 v
j


Step 4: Since all d
ij
is positive, therefore, the solution is optimal.

Transportation 4.2.12
[CWA INTER D02]
Q7: Comprehensive Formulation and Profit Maximization Problem
A Company has 4 factories F1, F2, F3and F4, manufacturing the same product. Production and raw
material costs differ from factory to factory and are given in the table below in the first two rows. The
transportation costs from the factories to the sales depots S1, S2 and S3 are also given. The last two
columns in the table below give the sales price and total requirements at each depot and the
production capacity of each factory is given in the last row.

F
1
F
2
F
3
F
4
S.P/p.u. () Requirement
Production Cost/Unit () 15 18 14 13
Raw Material Cost/Unit () 10 9 12 9
Transportation Cost/Unit ()
S
1
3 9 5 4 34 80
S
2
1 7 4 5 32 120
S
3
5 8 3 6 31 150
Production capacity 10 150 50 100

Determine the optimal solution and the associated profit by using the Vogels Approximation Method
A7: Profit matrix
Profit of each cell = Sale price (Production cost + Raw material cost + Transportation cost)
Maximization model

From\To S
1
S
2
S
3
A
F
1
6 6 1 10
F
2
(2) (2) (4) 150
F
3
3 2 2 50
F
4
8 5 3 100
Dummy 0 0 0 40
R 80 120 150 350

From\To S
1
S
2
S
3
A RP1 RP2 RP3 RP4
F
1
(2) 10(2) (7) 10 0 5
2
- -
F
2
(10) (10) (12) 150 0 2 2
F
3
(5) (6) (6) 50 1 0 0 1
F
4
80(0) 20(3) (5) 100 3
1
2 2 -
Dummy (8) (8) (8) 40 0 0 0 0
R 80 120 150 350
CP1 2 1 1
CP2 - 1 1

Resource Management 4.2.13
CP3 - 3
3
1
CP4 - 2 2

Q8: Unbalanced and Profit Maximization Problem
Departmental store wishes to purchase the following quantities of Sprees:

Types A B C D E
Quantity 150 100 75 250 200

Tenders are submitted by 4 different manufacturers who undertake to supply not more than the
quantities mentioned below (all types of sprees combined):

Manufacturer W X Y Z
Total Quantity 300 250 150 200

The store estimates that its profit/spree will vary with the manufacturer as shown in the following
matrix.
Sprees
Manufacturers A B C D E
W 275 350 425 225 150
X 300 325 450 175 100
Y 250 350 475 200 125
Z 325 275 400 250 175
How should the orders be placed?

Q9: Unbalanced and Minimization Problem with Alternative Solution
The products of three plants F1, F2 and F3 are to be transported to 5 ware houses W1, W2, W3, W4
and W5. The capacities of plants, demand of warehouses and the cost of transportation from one plant
to various warehouses are indicated in the following table:

W1 W2 W3 W4 W5 Plant Capacity
F1 74 56 54 62 68 400
F2 58 64 62 58 54 500
F3 66 70 52 60 60 600
Warehouse Demand 200 280 240 360 320 1500/1400

a) Find out a distribution plan of products from plants to the warehouses at a minimum cost. What
is the minimum cost?
b) Is there any surplus capacity of the plants? If so, in which plant should we associate that surplus
capacity?

Transportation 4.2.14
c) Is there any alternate solution for the optimum solution achieved in?

Q10: Basic Formulation, Unbalanced and Minimization Problem
The Bombay Transport Company has trucks available at four different sites in the following numbers:

Site A 5 Trucks
Site B 10 Trucks
Site C 7 Trucks
Site D 3 Trucks

Customers W, X and Y require trucks as shown below.

Customer W 5 Trucks
Customer X 8 Trucks
Customer Y 10 Trucks

Variable Costs of getting trucks to the Customers are given below:
From A to W 7, to X 3, to Y 6
From B to W 4, to X 6 to Y 8
From C to W 5, to X 8 to Y 4
From D to W 8 to X 4 to Y 3
Solve the above transportation problem.

Vous aimerez peut-être aussi